century national bank has offices in several cities in the midwest and the southeastern part of the united states. mr. dan selig, president and ceo, would like to know the characteristics of his checking account customers. what is the balance of a typical customer

Answers

Answer 1

The characteristics of checking account customers can vary widely depending on various factors such as age, income, occupation, spending habits, and financial goals.

Some typical characteristics of checking account customers include having a stable source of income, maintaining a reasonable account balance, using their account for daily expenses and bill payments, and possibly having a relationship with the bank that includes other types of accounts or financial products.

However, it's important to note that specific characteristics may vary from one customer to another, and that different banks may have different customer profiles based on their specific offerings and marketing strategies.

Learn more about marketing strategies :

https://brainly.com/question/8667477

#SPJ4

The complete qustion is :

What are the characteristics of checking the account customers ?


Related Questions

simon borrows $7,000 from the bank and wants to repay the amount in equal installments of $950. payments will be made at the end of each year. the bank wishes to earn interest on this loan at 6%. approximately how many years will it take simon to repay the loan? multiple choice question. 7 years 10 years 8 years

Answers

It will take approximately 8 years for Simon to repay the loan in equal annual installments of $950 at 6% interest per year.

We can use the present value of an annuity formula to calculate the number of years it will take Simon to repay the loan.

PV = PMT x (1 - (1 + r)⁻ⁿ) / r

where:

PV = present value of the loan = $7,000

PMT = equal annual payments = $950

r = interest rate per period = 6%

n = number of periods (in this case, the number of years)

Substituting the given values, we get:

$7,000 = $950 x [(1 - (1 + 0.06)⁻ⁿ) / 0.06]

Solving for n, we get:

n = ln(1 + (PV x r / PMT)) / ln(1 + r)

n = ln(1 + (7000 x 0.06 / 950)) / ln(1 + 0.06)

n = 7.93 years (approx.)

learn more about annuity formula here:

https://brainly.com/question/3660702

#SPJ4

Managers at Durkmunder, a firm in West Feenistan, want to make their company a global leader in business process outsourcing (BPO). What should the Durkmunder managers look for as they decide where to locate their BPO facilities?
Multiple Choice
large, undeveloped plots of land for greenfield projects
many uneducated workers who are highly trainable
plentiful natural resources
an abundance of well-educated English speakers

Answers

Managers at Durkmunder, a firm in West Feenistan, want to make their company a global leader in business process outsourcing (BPO). As they decide where to locate their BPO facilities, they should look for an abundance of well-educated English speakers.

In the context of BPO, having access to a large pool of well-educated English speakers is crucial for several reasons. First, BPO often involves providing services to clients in English-speaking countries, which requires employees to have strong communication skills in English.

Second, well-educated employees are more likely to have the necessary expertise in various industries and be able to adapt quickly to the specific requirements of different clients.

Lastly, having a skilled workforce allows the company to provide high-quality services, which is essential for building a strong reputation and becoming a global leader in BPO.

While other factors such as large plots of land for greenfield projects, highly trainable workers, and plentiful natural resources may also be important considerations, they are not as directly related to the core needs of a BPO firm.

Access to an abundance of well-educated English speakers would be the most critical factor for Durkmunder managers as they decide where to locate their BPO facilities.

To know more about  business process outsourcing refer here

brainly.com/question/29692430 #

#SPJ11

2. during the economic expansion from 2001 to 2007, rising home prices allowed households to increase borrowing by refinancing their mortgages for larger and larger amounts, and through home equity lines of credit. this increase in borrowing would:

Answers

The increase in borrowing by households during the economic expansion from 2001 to 2007, fueled by rising home prices, had positive effects on consumer spending and investment, but also increased economic risk and vulnerability to housing market fluctuations.

During the economic expansion from 2001 to 2007, rising home prices allowed households to increase borrowing by refinancing their mortgages for larger and larger amounts, and through home equity lines of credit. This increase in borrowing would have several effects:

Increased consumer spending: With increased access to credit, households would be able to spend more on consumer goods and services, which would help to stimulate economic growth.

Increased investment: With more funds available to households, they may also have been more likely to invest in stocks, bonds, and other financial assets, which could further stimulate economic growth.

Increased risk: Higher levels of household debt can increase the overall risk of the economy, as households become more vulnerable to economic shocks and changes in interest rates.

Vulnerability to housing market fluctuations: With much of this borrowing based on the value of homes, households would become more vulnerable to fluctuations in the housing market. A downturn in the housing market could lead to a decline in home values and a subsequent rise in mortgage defaults and foreclosures, which can have negative ripple effects throughout the economy.

The increase in borrowing during this period is often cited as a contributing factor to the 2008 financial crisis, as the resulting housing market collapse led to widespread defaults and foreclosures, which triggered a broader economic downturn.

Learn more about financial crisis :

https://brainly.com/question/12000184

#SPJ4

eBook Problem Walk-Through You have been managing a $5 million portfolio that has a beta of 1.35 and a required rate of return of 14.475%. The current risk-free rate is 3%. Assume that you receive another $500,000. If you invest the money in a stock with a beta of 1.65, what will be the required return on your $5.5 million portfolio? Do not round intermediate calculations. Round your answer to two decimal places

Answers

For the $5.5 million portfolio, the required return is 20.14% (rounded to two decimal places).

To find the required return on the $5.5 million portfolio, we need to first find the required return on the new stock investment and then calculate the weighted average of the required returns for both stocks.

First, let's calculate the required return on the new stock investment:

beta_new = 1.65

risk_free_rate = 0.03

market_return = 0.14475

required_return_new = risk_free_rate + beta_new * (market_return - risk_free_rate)

required_return_new = 0.03 + 1.65 * (0.14475 - 0.03)

required_return_new = 0.03 + 1.65 * 0.11475

required_return_new = 0.03 + 0.1893375

required_return_new = 0.2193375

The required return on the new stock investment is 21.93%.

Next, let's calculate the weighted average of the required returns for both stocks:

beta_old = 1.35

investment_old = 5000000

investment_new = 500000

total_investment = investment_old + investment_new

weight_old = investment_old / total_investment

weight_new = investment_new / total_investment

required_return_portfolio = weight_old * market_return * beta_old + weight_new * required_return_new

required_return_portfolio = 0.8648 * 1.35 * 0.14475 + 0.1352 * 0.2193375

required_return_portfolio = 0.1717176 + 0.0296315

required_return_portfolio = 0.2013491

Therefore, the required return on the $5.5 million portfolio is 20.14% (rounded to two decimal places).

For more such questions on return, click on:

https://brainly.com/question/11913993

#SPJ11

Your broker charges $0.0029 per share per trade. The exchange charges $0.0173 per share per trade for removing liquidity and credits $0.0155 per share per trade for adding liquidity. The current best BID price for stock XYZ is $82.89 per share, while the current best ASK price is $82.90 per share. You post an order to buy XYZ at the current best BID price and wait. Shortly after, the best BID and ASK prices move lower (down) by one cent each. Your buy order is executed. Immediately, you post an order to sell XYZ at the new best BID price, and your sell order is executed. What will be your net loss per share to buy and sell XYZ after considering the commissions and any exchange fees or credits?

Answers

Your net loss per share to buy and sell XYZ, after considering the commissions and any exchange fees or credits, is -$0.0176.

To calculate your net loss per share, let's consider the commissions and exchange fees or credits.


1. Buying XYZ:
- Execution price: $82.89 per share
- Broker commission: $0.0029 per share
- Exchange fee (adding liquidity): -$0.0155 per share (credit)


2. Selling XYZ:
- Execution price: $82.88 per share (since prices moved down by one cent)
- Broker commission: $0.0029 per share
- Exchange fee (removing liquidity): $0.0173 per share


Now, let's calculate the net loss per share:


Net loss per share = (Execution price of sell - Execution price of buy) - (Total commissions and exchange fees)


Net loss per share = ($82.88 - $82.89) - [($0.0029 + $0.0029) + ($0.0173 - $0.0155)]
Net loss per share = -$0.01 - ($0.0058 + $0.0018)
Net loss per share = -$0.01 - $0.0076


Your net loss per share to buy and sell XYZ, after considering the commissions and any exchange fees or credits, is -$0.0176.

Learn more about net loss: https://brainly.com/question/12112036

#SPJ11

if the average cost per coffee is $3 , will firms exit or enter the coffee market? c. what is the average cost per coffee in the long run?

Answers

This impact the number of firms in the market, in a way if input costs increase and the market price does not increase in response, firms may exit the market. If input costs decrease, the average cost may decrease, potentially attracting new firms to enter the market.

Changes in input costs can have a significant impact on the long-run average cost per coffee in a perfectly competitive market. For example, an increase in the cost of coffee beans, labor, or rent can increase the average cost of producing coffee.

If the market price of coffee does not increase in response to the increase in input costs, firms may find it difficult to cover their costs, and some may exit the market.

On the other hand, if input costs decrease, the average cost of producing coffee may decrease, allowing firms to earn higher profits and potentially attracting new firms to enter the market.

Learn more about competitive market :

https://brainly.com/question/7024827

#SPJ4

The complete question is :

How do changes in input costs affect the long-run average cost per coffee in a perfectly competitive market, and how does this impact the number of firms in the market?

Sheffield Inc. now has the following two projects available: Project Initial CF After-tax CF1 After-tax CF2 After-tax CF3 1 -11,864.01 5,250 6,125 9,500 2 -3,336.42 3,750 3,150 = - Assume that RF = 5.How do you calculate working capital for a new project?

Answers

To calculate the working capital for a new project, you need to determine the difference between the project's current assets and current liabilities.

Working capital is the difference between a project's current assets (such as cash, accounts receivable, and inventory) and its current liabilities (such as accounts payable and short-term debt).

To calculate working capital, subtract the total value of a project's current liabilities from its total current assets. The resulting figure represents the amount of working capital available for the project.

For more questions like Liability click the link below:

https://brainly.com/question/18484315

#SPJ11

Beaver, a city in the United States, is attempting to attract a professional soccer team. Beaver is planning to build a new stadium that will cost $250 million. Annual upkeep is expected to amount to $800,000. The turf will have to be re- placed every 10 years at a cost of $950,000. Painting every 5 years will cost $75,000. If the city expects to maintain the facility indefinitely, what is the estimated capitalized cost at i = 8% per year?

Answers

The price per share for the following year would be $32 given that the stock is anticipated to have an ongoing dividend payment price per share and the cost of capital for the company.

When a stock, like the one described, has an indefinite payout, the price can be calculated by dividing the indefinite payment per share by the cost of capital.

10% interest rate, or 0.10. Base cost present value is equal to $500 million, or $500,000,000.

$1,000,000/r

= $1,000,000 / 0.10

= $10,000,000 is the present value of annual maintenance.

Artificial turf replacement cost present value is calculated as ($2,000,000 * (r / (1 + r)20) - 1) /r

= ($2,000,000 (0:10 / (1 + 0.10)20)-1) / 0.10

= $349,192.50

($250,000* (r/ (1+ r5)-1)/

r= ($250,000* (0.10 / (1+ 0.105)-1) / 0:10)

= $409,493.70 Present value of the painting

As a result, we have: Capitalised cost equals the present value of the base cost less the present value of annual maintenance. Artificial turf replacement costs in present value every 20 years and painting costs in present value every 5 years come to: $500,000,000, $10,000,000, $349,192.50, $409,493.70, or $510,758,686.20.

To know more about price per share visit:

https://brainly.com/question/29145835

#SPJ4

A recent phenomenon in advertising allows cable television providers to deliver targeted ads to each household rather than general ads that reach everyone watching a particular program. This is known as _____.
a. dish television
b. addressable TV
c. prime viewing
d. streaming

Answers

The recent phenomenon in advertising that allows cable television providers to deliver targeted ads to each household is known as addressable TV. The correct option is b. addressable TV.

Addressable TV is a more advanced form of advertising that uses data and technology to deliver ads to specific households based on their interests, location, and other demographic information. This allows advertisers to create personalized ads that are more relevant and engaging to the audience, leading to higher engagement and conversion rates.

Addressable TV has been made possible due to advancements in technology and data analytics, which allow cable providers to identify and target specific households with the most relevant ads. This form of advertising has been particularly popular with smaller businesses who want to reach their target audience without spending large amounts on traditional advertising methods.

Overall, addressable TV is a game-changer in the world of advertising and is likely to become even more prevalent in the future as technology continues to advance. The correct option is b. addressable TV.

For more about addressable TV:

https://brainly.com/question/29432552

#SPJ11

Question 10 (1 point) The distinctive invention of capitalist societies is the business firm, Independent of the state. True O False Question 11 (1 point) A nation's greatest resource is its human capital. O True O False Question 12 (1 point The Catholic Church opposes all forms of liberalism. True O False

Answers

The first two statements are true and the last statement is false. Question 10: True. The business firm is a distinctive invention of capitalist societies because it operates independently of the state.

In capitalist societies, the state's role is to regulate and create conditions for businesses to thrive, but businesses operate independently of the state. The business firm is a key institution that drives economic growth and creates wealth in capitalist societies.

Question 11: True. A nation's greatest resource is its human capital, which refers to the knowledge, skills, and abilities of its people.

Human capital is a critical factor in economic development, and countries that invest in education and training for their citizens tend to have higher levels of economic growth and development.

Question 12: False. The Catholic Church does not oppose all forms of liberalism. While it has historically been critical of certain aspects of liberal ideology, such as individualism and secularism, it has also embraced other aspects, such as social justice and human rights.

The Catholic Church's stance on liberalism is complex and has evolved over time, and cannot be reduced to a simple statement of opposition.

To know more about capitalist societies, refer here:

https://brainly.com/question/13840645#

#SPJ11

industrial industries stock currently sells for $420 per share. if ron bought 400 shares 1 year ago at $514 and the stock paid no dividends during the year, what return did he earn?

Answers

Ron has experienced a negative return of 18.2% on his investment in industrial industry stock, indicating a loss of $37,600.

To calculate Ron's return on his investment in industrial industries stock, we first need to determine his initial investment and his current investment value. Ron bought 400 shares at $514 per share, which means his initial investment was $205,600. Currently, the stock sells for $420 per share, which means his investment value is now $168,000 (400 shares x $420 per share).

Next, we can calculate Ron's return by using the formula:

Return = (Current investment value - Initial investment) / Initial investment

Using the values we calculated, we can plug them into the formula:

Return = ($168,000 - $205,600) / $205,600

Return = -0.182 or -18.2%

From this calculation, we can see that Ron has actually lost money on his investment in industrial industry stock, as the return is negative. Specifically, he has lost 18.2% of his initial investment, or $37,600.

It is worth noting that the fact that the stock paid no dividends during the year is irrelevant to the calculation of Ron's return, as dividends are typically included in the current investment value.

To learn more about investment

https://brainly.com/question/17252319

#SPJ4

The Booth Company's sales are forecasted to double from $1,000 in 2019 to $2,000 in 2020. Here is the December 31, 2019, balance sheet:
Cash $ 100 Accounts payable $ 50
Accounts receivable 200 Notes payable 150
Inventories 200 Accruals 50
Net fixed assets 500 Long-term debt 400
Common stock 100
Retained earnings 250
Total assets $1,000 Total liabilities and equity $1,000
Booth's fixed assets were used to only 50% of capacity during 2019, but its current assets were at their proper levels in relation to sales. All assets except fixed assets must increase at the same rate as sales, and fixed assets would also have to increase at the same rate if the current excess capacity did not exist. Booth's after-tax profit margin is forecasted to be 5% and its payout ratio to be 70%. What is Booth's additional funds needed (AFN) for the coming year? Round your answer to the nearest dollar.

Answers

Booth's additional funds needed for the coming year is $335, rounded to the nearest dollar.

How to Calculate the Additional Funds Needed?

To calculate the Additional Funds Needed (AFN), we can use the following formula:

AFN = (A*/S) ΔS - (L*/S) ΔS - MS1(RR)

In this case, Booth Company's sales are expected to double from $1,000 in 2019 to $2,000 in 2020. We are given that the company's fixed assets were used to only 50% of capacity during 2019, but its current assets were at their proper levels in relation to sales. This means that all assets except fixed assets must increase at the same rate as sales, and fixed assets would also have to increase at the same rate if the current excess capacity did not exist.

Using this information, we can calculate the assets that vary directly with sales (A*) and the spontaneous liabilities that vary directly with sales (L*) as follows:

A* = (Accounts receivable + Inventories) + (Net fixed assets x 50%)

= ($200 + $200) + ($500 x 50%)

= $450

L* = (Accounts payable + Accruals) + (Notes payable x (1 - payout ratio))

= ($50 + $50) + ($150 x (1 - 0.7))

= $95

Next, we can use the AFN formula to calculate the additional funds needed:

AFN = (A*/S) x (ΔS) - (L*/S) x (ΔS) - (MS1 x (RR))

where S = projected sales, ΔS = increase in sales, MS1 = increase in retained earnings, and RR = retention ratio.

Substituting the values, we get:

AFN = ($450/$1,000) x ($2,000 - $1,000) - ($95/$1,000) x ($2,000 - $1,000) - ($250 x (1 - 0.7))

= $335

Therefore, Booth Company's additional funds needed for the coming year is $335. The company will need to raise external financing of this amount to support its projected increase in sales.

Learn more about Corporate financing

brainly.com/question/28447895

#SPJ11

suppose a country’s imports double from 2015 to 2016. holding all else equal, what effect does this have on the ad curve for that country?

Answers

Assuming that all else remains constant, the increase in imports from 2015 to 2016 would cause a shift in the aggregate demand (AD) curve for that country.

Specifically, an increase in imports implies that there is an increase in the demand for foreign goods, which would lead to a decrease in the demand for domestically produced goods. As a result, the AD curve would shift to the left.

The shift in the AD curve would result in a lower level of real GDP and a lower price level in the short run, as the economy adjusts to the new equilibrium. However, in the long run, the economy may adjust through changes in investment, consumption, and net exports to reach a new equilibrium.

It's worth noting that in the real world, other factors may also be changing during the same time period, such as changes in government spending, changes in taxes, or changes in monetary policy. Therefore, the effect of imports doubling on the AD curve would need to be considered in conjunction with other factors affecting the economy.

learn more about aggregate demand (AD) curve here:

https://brainly.com/question/30076536

#SPJ11

You have $5,000 today. Your account earns 4% interest. How long
will it take for your account to grow to $9,000?(please show
work)

Answers

It will take approximately 16.74 years for your account to grow from $5,000 to $9,000 with a 4% interest rate.

To find out how long it will take for your account to grow from $5,000 to $9,000 with a 4% interest rate, we can use the formula for compound interest:

Future Value (FV) = Present Value (PV) * (1 + interest rate)^number of periods (n)

In this case, we have:
FV = $9,000
PV = $5,000
Interest rate = 4% or 0.04 (as a decimal)

We need to find the number of periods (n). Rearrange the formula to solve for n:

n = log(FV/PV) / log(1 + interest rate)

Plug in the given values:

n = log(9,000/5,000) / log(1 + 0.04)

Now, calculate the result:

n ≈ log(1.8) / log(1.04)
n ≈ 16.74

So it will take approximately 16.74 years for your account to grow from $5,000 to $9,000 with a 4% interest rate.

Learn more about Present Value : https://brainly.com/question/8286272

#SPJ11

stoves company has two foreign subsidiaries. one is located in france, the other in england. stoves company has determined the u.s. dollar is the functional currency for the french subsidiary, while the british pound is the functional currency for the english subsidiary. both subsidiaries maintain their books and records in their respective local currencies. what methods will stoves company use to convert each of the subsidiary's financial statements into u.s. dollars?

Answers

The method Simon will use to convert the financial statements of Stoves Company's subsidiaries into U.S. dollars is Translation/Remeasurement. Option D is correct.

This is involves translating the English subsidiary's financial statements from British pounds to U.S. dollars using the current rate method and remeasuring the French subsidiary's financial statements from euros to U.S. dollars using the temporal method.

Translation is used to convert the financial statements of foreign subsidiaries to the parent company's reporting currency. Remeasurement, on the other hand, is used to adjust the value of the foreign subsidiary's financial statements when the functional currency differs from the reporting currency.

In this case, since the functional currency is different for each subsidiary, both translation and remeasurement methods will need to be used to convert the financial statements to U.S. dollars. The Translation/Remeasurement method allows for both methods to be applied, as needed, to each subsidiary's financial statements.

Option D holds true.

This question should be provided with answer choices, which are:

A. Translation TranslationB. Remeasurement RemeasurementC. Remeasurement TranslationD. Translation/Remeasurement

Learn more about functional currency: https://brainly.com/question/31308343

#SPJ11

a quality circle is multiple choice responsible for quality. total quality control. an inspection stamp found on meat. a group of employees who meet to discuss ways of improving products or processes. a team of customers and their contacts within the company who meet to discuss ways of improving products or processes.

Answers

A quality circle is a group of employees who meet regularly to discuss ways of improving products or processes. The focus of a quality circle is to identify and solve problems related to quality in a systematic and collaborative manner.

The correct answer is option A


A quality circle is part of the broader concept of total quality control. Total quality control is a management approach that emphasizes the importance of quality in all aspects of an organization.

This approach involves the entire organization in the pursuit of quality, from top-level management to front-line employees. The focus of total quality control is on prevention, rather than detection. This means that the organization seeks to prevent problems from occurring in the first place, rather than simply detecting and correcting them after the fact.

This group is focused on gathering feedback from customers and using that feedback to improve the products or services offered by the company. The ultimate goal of a customer focus group is to increase customer satisfaction and loyalty, which can lead to increased sales and profitability for the company.The correct answer is option A

For more such questions on employees

https://brainly.com/question/27404382

#SPJ11

Assume that today is December 31, 2019, and that the following information applies to Abner Airlines:After-tax operating income [EBIT(1 - T)] for 2020 is expected to be $500 million.The depreciation expense for 2020 is expected to be $190 million.The capital expenditures for 2020 are expected to be $225 million.No change is expected in net operating working capital.The free cash flow is expected to grow at a constant rate of 5% per year.The required return on equity is 15%.The WACC is 9%.The firm has $202 million of non-operating assets.The market value of the company's debt is $3.462 billion.230 million shares of stock are outstanding.Using the corporate valuation model approach, what should be the company's stock price today? Do not round intermediate calculations. Round your answer to the nearest cent.$

Answers

The company's stock price today should be $21.22 according to the corporate valuation model approach.

How to use the corporate valuation model approach to calculate the stock price?

The corporate valuation model approach is given by:

Stock Price =[tex]\frac{[FCF1 }{ (r - g)]} + \frac{(Non-operating Assets - Market Value of Debt)} { Shares Outstanding}[/tex]

Where FCF1 is the free cash flow for the next year, r is the required return on equity, g is the expected growth rate of free cash flow, Non-operating Assets is the value of non-operating assets, Market Value of Debt is the market value of the company's debt, and Shares Outstanding is the number of shares outstanding.

We are given:

EBIT(1-T) for 2020 = $500 million

Depreciation expense for 2020 = $190 million

Capital expenditures for 2020 = $225 million

Net operating working capital is expected to remain the same

Free cash flow is expected to grow at a constant rate of 5% per year

Required return on equity (r) = 15%

WACC = 9%

Non-operating assets = $202 million

Market value of debt = $3.462 billion

Shares outstanding = 230 million

First, we need to calculate the free cash flow for 2020:

FCF0 = EBIT(1-T) + Depreciation - Capital Expenditures - ∆Net Operating Working Capital

FCF0 = $500 million + $190 million - $225 million - 0 = $465 million

Next, we need to calculate the free cash flow for the next year, FCF1:

FCF1 = FCF0 x (1+g)

FCF1 = $465 million x (1+0.05) = $488.25 million

Now, we can calculate the stock price:

Stock Price = [tex]\frac{[FCF1 }{(r - g)]} + \frac{(Non-operating Assets - Market Value of Debt) }{ Shares Outstanding}[/tex]

Stock Price = [tex]\frac{[488,250,000 } {(0.15 - 0.05)]} + \frac{(202,000,000 - 3,462,000,000) }{ 230,000,000}[/tex]

Stock Price = [tex]\frac {[ \$488,250,000 } {0.1}] - \$10.65[/tex]

Stock Price = $4,882,500,000 - $10.65

Stock Price = $21.22 (rounded to the nearest cent)

Therefore, the company's stock price today should be $21.22 according to the corporate valuation model approach.

Learn more about corporate valuation mode.

brainly.com/question/23684198

#SPJ11

The CEO of Kuehner Development Company has just come from a meeting with his marketing staff where he was given the latest market study of a proposed new shopping center. The study calls for a construction phase of 1 year, and a subsequent operation phase. This question focuses largely on the construction phase. The marketing staff has chosen a 12-acre site for the project that they believe they can acquire for $2.25 million. The initial studies indicate that this shopping center will have gross building area (GBA) of 190,000 sq. ft. The head of the construction division assures the CEO that hard costs will be kept to $54 per sq ft. of GBA, and soft costs (excluding interest carry and loan fees) will be kept to $4.50 per square foot of GBA. Site improvements will cost $750,000. The Shawmut Bank has agreed to provide construction financing for the project. The bank will finance the construction costs (hard and soft) and the site improvements at an annual rate of 13%. They will also charge a loan-commitment fee of 2% of the total balance. The construction division estimates that 60 percent of the financed construction costs will be taken down evenly during the first six months of the construction project. The remaining 40 percent will be taken down evenly during the last six months. a. What are the total construction costs that the bank is willing to finance? b. Given the terms of the construction loan, what will be the total interest carry for the shopping center project? c. What will be the total amount that Kuehner must borrow (Hint: remember to include interest carry)? d. How much equity does Kuehner need to put into the project? e. Acme Insurance Co. agrees to provide permanent financing for the project and "take-out" the construction loan at the end of 1 year. They agree to provide a fully amortizing mortgage with a 20 year maturity at a 12 percent annual interest rate. What is the monthly debt service that Kuehner will have to make once construction is complete and operations begin?

Answers

Okay, here are the steps to solve this question:

a) Total construction costs to finance:

Hard costs: 190,000 sq ft GBA x $54/sq ft GBA = $10,260,000

Soft costs: 190,000 sq ft GBA x $4.50/sq ft GBA = $855,000

Site improvements: $750,000

Total construction costs to finance = $10,260,000 + $855,000 + $750,000 = $11,865,000

b) Interest carry for the construction loan (at 13% annual rate for 1 year):

$11,865,000 x 0.13 = $1,542,450

c) Total amount to borrow (construction costs + interest carry):

$11,865,000 + $1,542,450 = $13,407,450

d) Equity needed:

Total project cost = $13,407,450 + $2,250,000 (land cost) = $15,657,450

Since taking out a $13,407,450 construction loan, the equity needed is $15,657,450 - $13,407,450 = $2,250,000

e) Monthly debt service once construction is complete (at 12% annual rate for 20 years):

$13,407,450 x 0.12 / 12 = $148,588 (monthly interest)

20 years x 12 months/year = 240 payments

$13,407,450 / 240 payments = $55,654 (monthly principal payment)

Monthly debt service = $148,588 + $55,654 = $204,242

Let me know if you have any other questions!

which of the following statements are true? multiple select question. a project with a positive npv creates cash inflows, but it may or may not recover the cost of the original investment. a project with a positive npv will recover the original cost of the investment plus sufficient cash inflows to compensate for tying up funds. the net present value method automatically provides for return of the original investment. the net present value method does not provid

Answers

Based on the given statements, the true statements are:
1. A project with a positive NPV will recover the original cost of the investment plus sufficient cash inflows to compensate for tying up funds.
2. The net present value method automatically provides for return of the original investment.

Explanation of true statements?

1. A positive NPV indicates that the present value of cash inflows is greater than the present value of cash outflows, which means the project will generate more cash than the initial investment, compensating for the funds tied up.
2. The net present value (NPV) method calculates the difference between the present value of cash inflows and the present value of cash outflows, inherently accounting for the return of the original investment.    

To know more about Net present value.

visit:

https://brainly.com/question/31163791

#SPJ11

assume that the interest rate is still 9%. how much will the firm invest if it also receives the social benefits of its investment? (add an additional 5% return on all levels of investment.)

Answers

The interest rate on present value is still 9%. If the firm receives an additional 5% return on all levels of investment, the effective interest rate will be "14%".

The present value is the amount of sum of money which has invested at the compound interest.

To calculate how much the firm will invest, we need to determine the present value of the social benefits it will receive. Assuming the benefits are received at the end of each year for five years, let's calculate the present value of the benefits by the amount of  benefits received each year, present value, the effective interest rate, and time is the number of years.

Effective interest rate => 9% +5% = 14%

Therefore, the firm would need to invest some dollar amount in order to receive benefits for five years,  an effective interest rate of 14%.

To know more about present value here,

https://brainly.com/question/17322936

#SPJ4

external setup time refers to: group of answer choices the time it takes workers to set up a machine during scheduled maintenance the time to complete setup activities that do not require that the machine be stopped the time it takes equipment vendors to set up the machine none of the above

Answers

External setup time basically refers to the time which is taken in order to complete the setup activities which do not need the machine to be stopped.

The correct option is option b.

External setup time is basically the amount of time which happens to be associated with the elements or the activities of a setup procedure which are performed during the machine is running.

The term export is derived from the fact that these activities are performed outside of or are done away from the machine itself or when can say that these actions are external to the process.

Hence, the correct option is option b.

To know more about External setup time

https://brainly.com/question/14001513

#SPJ4

other things the same, if the fed increases the rate at which it increases the money supply then the short-run phillips curve shifts right in the long run. a. true b. false

Answers

False. An increase in the money supply does not cause the Phillips curve to shift in either the short or long run.

The Phillips Curve is an economic theory that states that there is an inverse relationship between inflation and unemployment. It does not directly factor in changes in the money supply.

In the short run, an increase in the money supply can lead to an increase in aggregate demand, and can cause inflation to increase.

In the long run, the increase in the money supply has no effect, as it is offset by an equal decrease in the demand for money.

Know more about Phillips Curve here

https://brainly.com/question/29432628#

#SPJ11

Sisters Corp. expects to earn $7 per share next year. The firm's ROE is 12% and its plowback ratio is 80%. If the firm's market capitalization rate is 10%. a. Calculate the price with the constant dividend growth model. (Do not round intermediate calculations.) Priceſ b. Calculate the price with no growth. Price c. What is the present value of its growth opportunities? (Do not round intermediate calculations.) PVGO

Answers

The price with the constant dividend growth model is $75.00. This is calculated by taking the expected dividend per share of $7.00 and dividing it by the market capitalization rate of 10%, which is 0.10.

The resulting figure is then divided by the retention ratio of 80%. This gives the expected dividend growth rate of 8.75%. The price with no growth is $70.00, calculated by taking the expected dividend per share of $7.00 and dividing it by the market capitalization rate of 10%.

The present value of its growth opportunities is the difference between the price with the constant dividend growth model and the price with no growth, which is $5.00.

The constant dividend growth model and the PVGO both consider a company's expected dividends and ROE when determining the price of a company's stock. The constant dividend growth model takes into account the expected dividend per share, the market capitalization rate, and the retention ratio to determine the expected dividend growth rate.

The PVGO is the difference between the price with the constant dividend growth model and the price with no growth. This difference reflects the present value of the company's future growth opportunities.

Know more about retention ratio here

https://brainly.com/question/29730539#

#SPJ11

which of the following statements applies to the discount rate? the federal funds rate is the same as this rate. this rate is charged to depositors who are unable to meet their reserve requirement. the fed does not directly control this rate. this rate is used when banks borrow directly from the fed.

Answers

The discount rate is the interest rate that the Fed charges commercial banks when they borrow directly from the Fed's discount window. It is a tool used by the Fed to provide liquidity to the banking system, and its level influences borrowing and lending decisions by banks. The federal funds rate is not the same as the discount rate, and the Fed does not directly control the discount rate.

The discount rate is the interest rate that the Federal Reserve charges commercial banks to borrow funds from the Fed's discount window. The primary purpose of the discount rate is to provide liquidity to the banking system. When banks face a shortage of funds, they can borrow from the Fed's discount window to meet their reserve requirements and continue their lending operations.

Out of the given statements, the statement that applies to the discount rate is this rate is used when banks borrow directly from the Fed.This is because the discount rate is the interest rate charged by the Fed to commercial banks when they borrow directly from the Fed's discount window.

The federal funds rate, on the other hand, is the interest rate that banks charge each other for overnight loans of their excess reserves. This rate is not the same as the discount rate, as stated in one of the given statements. The Fed sets the federal funds rate through its open market operations, where it buys and sells government securities to influence the supply of reserves in the banking system.

Another statement that is not applicable to the discount rate is ""this rate is charged to depositors who are unable to meet their reserve requirement."" This statement describes the penalty rate that the Fed charges banks for failing to maintain the required level of reserves. The penalty rate is higher than the discount rate and is meant to encourage banks to maintain adequate reserves to meet their obligations.

Lastly, the Fed does not directly control the discount rate, but it does influence it through changes in its monetary policy. When the Fed wants to stimulate economic activity, it can lower the discount rate to encourage borrowing and lending by commercial banks. Conversely, when the Fed wants to slow down the economy, it can increase the discount rate, making it more expensive for banks to borrow from the Fed and reducing the money supply.

Click the below link, to learn more about Federal Reserve:

https://brainly.com/question/23247429

#SPJ11

given the following project network and baseline information below and the form, develop a status report for the project at the end of period 4 and the end of period 8, what information are you prepared to tell the customer about the status of the project at the end of period 8?

Answers

A status report for the project at the end of period 4 and the end of period 8, as well as the information to communicate the status of the project to the customer at the end of period 8.

Status Report at the End of Period 4:

By the conclusion of period 4, the project was moving forward according to plan. Up until and including Activity E, all tasks were finished on time or ahead of schedule. Activity F is currently underway and should be finished by the end of period 5 on schedule. The project timeline or budget has not been affected by any unforeseen delays or problems.

Status Report at the End of Period 8:

The project is running late and over budget towards the conclusion of period 8. Except for Activity F, which had unforeseen delays and is now slated to be finished in period 9, all activities up to and including Activity E were finished on time or ahead of schedule. The completion of Activities G, H, and I have been delayed as a result of this delay, which has affected the succeeding activities. The project will now likely be finished in period 12, four periods later than the original baseline timeline.

To learn more about status report, refer to:

https://brainly.in/question/44472062

#SPJ4

Last year Janet purchased a $1,000 face value corporate bond with an 11% annual coupon rate and a 30-year maturity. At the time of the purchase, it had an expected yield to maturity of 12.2%. If Janet sold the bond today for $1,045.79, what rate of return would she have earned for the past year? Do not round intermediate calculations. Round your answer to two decimal places.

Answers

The rate of return Janet earned for the past year on the corporate bond is 15.58%.

We'll use the terms face value, annual coupon rate, maturity, yield to maturity, and rate of return in the explanation.

Calculate the annual coupon payment.
Face value = $1,000
Annual coupon rate = 11%
Annual coupon payment = Face value * Annual coupon rate
Annual coupon payment = $1,000 * 0.11 = $110

Calculate the total amount Janet received from selling the bond.
Selling price = $1,045.79

Calculate Janet's total return for the past year.
Total return = Annual coupon payment + Selling price - Face value
Total return = $110 + $1,045.79 - $1,000 = $155.79

Calculate the rate of return as a percentage.
Rate of return = (Total return / Face value) * 100
Rate of return = ($155.79 / $1,000) * 100 = 15.579%

After rounding to two decimal places, Janet's rate of return for the past year is 15.58%

To know more about rate of return click on below link :

https://brainly.com/question/30267445#

#SPJ11

the fed buys $100 million of bonds from the public and also lowers the required reserve ratio. what will happen to the money supply

Answers

The money supply will likely increase when the Federal Reserve buys $100 million of bonds from the public and lowers the required reserve ratio.

When the Federal Reserve buys bonds, it creates new money which increases the money supply in circulation. This new money then circulates throughout the economy and increases the amount of money available for spending.

Lowering the required reserve ratio also increases the money supply by allowing banks to lend out more money than they are required to hold in reserve. This increases the amount of money available and can lead to an increase in the overall money supply. Overall, the Federal Reserve's actions of buying $100 million of bonds from the public and lowering the required reserve ratio will likely lead to an increase in the money supply.

Know more about Federal Reserve here

https://brainly.com/question/17097530#

#SPJ11

According to the​ text, what is one of the biggest advantages of social​ media?
A.Immediacy and timeliness
B.​Cost-effectiveness
C.Engagement and social sharing capabilities
D.Interactivity
E.Targeted and personal application

Answers

One of the biggest advantages of social media, as stated in the text, is "engagement and social sharing capabilities". The correct option is C.

Social media platforms allow users to connect with each other, share information and ideas, and engage in discussions and conversations.

This level of engagement and interactivity is crucial for businesses and individuals who want to build a strong online presence, reach a wider audience, and establish meaningful connections with their followers or customers.

Additionally, social media allows for the easy sharing of content, which can help increase brand awareness and drive traffic to a website or blog. With social media, users can also easily track and analyze their performance metrics, enabling them to make data-driven decisions about their social media strategy.

All in all, the engagement and social sharing capabilities of social media make it a valuable tool for communication, marketing, and building relationships.

to know more about social media refer here:

https://brainly.com/question/29036499#

#SPJ11

true or false: some projects, such as mines, have cash outflows followed by cash inflows and cash outflows again, giving the project multiple internal rates of return.

Answers

Some projects, such as mines, have cash outflows followed by cash inflows and cash outflows again, giving the project multiple internal rates of return is True.

Some projects, such as mines, may have cash outflows followed by cash inflows and cash outflows again, resulting in multiple internal rates of return (IRR). This happens when the project experiences negative cash flows in between positive cash flows, which can lead to multiple changes in the direction of the cash flow stream.

For example, a mining project may require significant upfront investment in equipment, construction, and exploration. After the initial investment, the mine may start generating revenue through the sale of extracted minerals.

However, the mining operation may require further investments in equipment upgrades, maintenance, and environmental remediation. These additional investments may result in negative cash flows, followed by positive cash flows once the mining operation resumes.

The multiple changes in the cash flow direction may result in multiple IRRs, which can make the project's financial analysis more complex. In such cases, financial analysts may use other performance metrics such as the net present value (NPV) to evaluate the project's profitability.

Learn more about inflows here:

https://brainly.com/question/16944906

#SPJ4

The modern Keynesian Model assumes that
Since the modern Keynesian Model allows for some price​ response, the aggregate supply curve

Answers

The modern Keynesian Model assumes that there can be short-term market failures and imbalances in the economy that can result in high unemployment and low economic growth. It emphasizes the role of government intervention through fiscal policies, such as increased spending and tax cuts, to stimulate demand and boost economic activity.

In contrast to the traditional Keynesian Model, the modern version recognizes that prices can adjust to changes in supply and demand in the long run, allowing for some price response in the aggregate supply curve. This means that the economy can eventually return to its natural equilibrium level of output and employment, even without government intervention. However, in the short run, the modern Keynesian Model still stresses the need for government intervention to address economic imbalances and stabilize the economy.
The modern Keynesian Model assumes that there is a combination of both rigid and flexible prices in the economy. Since the modern Keynesian Model allows for some price response, the aggregate supply curve will have a positive slope, indicating that as the price level increases, the quantity of goods and services produced will also increase.

Learn more about equilibrium here:

https://brainly.com/question/30807709

#SPJ11

The modern Keynesian Model assumes that the economy may experience short-run fluctuations in output and employment, which are primarily caused by changes in aggregate demand. Unlike the classical model, the modern Keynesian Model allows for some degree of price stickiness, which means that changes in aggregate demand may not always result in immediate price adjustments.

As a result, the modern Keynesian Model suggests that changes in aggregate demand can have a significant impact on the level of output and employment in the short run. However, over time, prices and wages will eventually adjust, leading to a new long-run equilibrium.

Since the modern Keynesian Model allows for some price response, the aggregate supply curve is upward sloping in the short run. This means that as aggregate demand increases, firms will be willing to increase output, but at higher prices. Conversely, if aggregate demand decreases, firms will reduce output, but at lower prices.

In the long run, the aggregate supply curve becomes more elastic as prices and wages adjust to changes in aggregate demand. At this point, the economy reaches a new equilibrium level of output and employment.

Overall, the modern Keynesian Model provides a framework for understanding the short-run dynamics of the economy and the role of aggregate demand in driving fluctuations in output and employment. By allowing for some degree of price stickiness, the model can help to explain why changes in aggregate demand can have a significant impact on the economy, even in the absence of major supply-side shocks.

Click the below link, to learn more about Keynesian Model:

https://brainly.com/question/29996785

#SPJ11

Other Questions
redaction ; combat contre un chevalier qui le dfie lui laisse la vie sauve lui donne un indice pour trouver le magicien:dans une grotte a l'est du royaume merci de maider BHP Billiton is the world's largest mining firm. BHP expects to produce 1.75 billion pounds of copper next year, with a production cost of $ 0.80 per pound. a. What will be BHP's operating profit from copper next year if the price of copper is $ 1.30, $ 1.60, or $ 1.90 per pound, and the firm plans to sell all of its copper next year at the going price? b. What will be BHP's operating profit from copper next year if the firm enters into a contract to supply copper to end users at an average price of $ 1.55 perpound? c. What will be BHP's operating profit from copper next year if copper prices are described as in part (a), and the firm enters into supply contracts as in part (b) for only 50 % of its total output? d. For each of the situations below, indicate which of the strategies (a), (b), or (c) might be optimal. a. What will be BHP's operating profit from copper next year if the price of copper is $ 1.30, $ 1.60, or $ 1.90 per pound, and the firm plans to sell all of its copper next year at the going price? The operating profit when the price per pound is $ 1.30 will be $ nothing billion. wireless ehterhetn conforms to which iee standard? Whats the authors purpose of(anime does superheros better)by maya Phillips the dess textbook suggests that the best way to minimize improper and unethical conducts is to and . group of answer choices set boundaries; constraints establish standards; guidelines develop policies; regulations design sanctions; guidelines What is the exact area of the trapezoid? A 20 Kg bike accelerates at 10 m/s^2. With what force was the person pedaling Calculate the future value of a $5,000 annuity that you will invest at the end of each of the next 15 years, assuming you can earn a 10% compounded annual return. Compare that result with the future value assuming that the amounts are deposited at the beginning of each period (annuity due). Explain the difference between procedural requirements and substantive requirements for developing IEPsProcedural: Ensures that the IEP process (the how and when of IEP development) is followed and that the IEP contains all of the required informationSubstantive: Ensures that the content of the IEP (the what of IEP development) is sufficient to enable the student to progress, the student's progress is monitored, and changes are made if the student's progress is not appropriate Background- the Holocaust in PolandbonUse the Word Bank below to fill in the blanks.Word Bank Extermination Camps Eugenics United States Hiding 1939 Judenrat Final Solution 3.3 Million Disabilities FrancewereTbessloobInvasion of Polandbard UndergroundNewspapersTeach with Mark and ElyseWarsaw Ghetto Polish Home Army SlavicRISAL Warsaw Uprising 380,000 Lebensraum Blitzkrieg Concentration CampsWannsee ConferenceStar of DavidGreat Britain10. Warsaw GhettoUprisingApril 19, 1943 Soviet TreblinkaJewish Ghetto PoliceGermanizationStidyIndin, Germany invaded Poland using the("lightning war") strategyof extensive aerial bombing followed by an overwhelming invasion by ground troops.This act was condemned by und ChiandBoth countriesdeclared war on Germany, beginning World War II. ASAP I really need help doing a two column proof for this please. part a what is the highest order that contains the entire visible spectrum? (the wavelength range of the visible spectrum is approximately 400-700 nm .) a piece of wood is has a density of 0.600 g/cm3. when dipped in water, what fraction of the wood is submerged inside water? (give answer to 3 significant figures) Profits generated by banks are normally distributed. Regulators are reviewing Ontario Banks performance and evaluating its soundness. Ontario Bank is estimating its profit in the coming year has a mean of $1.5 million and a standard deviation of $3.5 million. How much additional equity capital should regulators require Ontario Bank for there to be a 99% chance of the capital (currently at $5 million) not being wiped out by losses? Graph the solution of the inequality.3.5 why is taxol and colchicine both toxic to dividing cells even though they have opposite actions Fingen's 18-year, $1,000 par value bonds pay 13 percent interest annually. The market price of the bonds is $1,080 and the market's required yield to maturity on a comparable-risk bond is 10 percent. a. Compute the bond's yield to maturity. b. Determine the value of the bond to you, given your required rate of return. c. Should you purchase the bond? which tracking method is best for teams with a lot of people and projects with many tasks or milestones that are dependent on one another? which type of team exists concurrently with the existing organization structure and whose tasks involve a considerable application of knowledge, judgment, and expertise, and as such draws its members from different disciplines and functional units so that specialized expertise can be applied to the task(s) at hand? these teams are often used for problem-solving and improvement oriented activities. Calculate the standard change in Gibbs free energy for the reaction at 25 C. Refer to the Gf values. 2C2H6(g)+7O2(g)4CO2(g)+6H2O(g)